Fix problem* vs. problem62 in Serway and Jewett v4's problem 1.62.
[course.git] / latex / problems / Serway_and_Jewett_4 / problem01.62.tex
1 \begin{problem*}{1.62}
2 In physics, it is important to use mathematical approximations.
3 Demonstrate that for small angles ($< 20\dg$)
4 \begin{equation}
5  \tan \alpha \approx \sin \alpha \approx \alpha = \pi \alpha ' / 180\dg
6 \end{equation}
7 where $\alpha$ is in radians and $\alpha '$ is in degrees.
8 Use a calculator to find the largest angle for which $\tan \alpha$ may 
9 be approximated by $\alpha$ with an error less than $10.0$\%.
10 \end{problem*}
11
12 \begin{solution}
13 To kill both birds with one stone,
14  a table to show the approximations hold
15   and show the \% error of the approximation:\\
16 \begin{tabular}{|r|r|r|r|r|}
17 \hline
18 $\alpha '$&$\alpha$ [rad]&$\sin \alpha$&$\tan \alpha$&\% error\\
19 \hline
20  $0\dg$&0.000&0.000&0.000&$\emptyset$\\
21  $5\dg$&0.087&0.087&0.087&$-0.25$\%\\
22 $10\dg$&0.175&0.174&0.176&$-1.02$\%\\
23 $15\dg$&0.262&0.259&0.268&$-2.30$\%\\
24 $20\dg$&0.349&0.342&0.354&$-4.09$\%\\
25 $31\dg$&0.541&0.515&0.601&$-9.95$\%\\
26 $32\dg$&0.599&0.530&0.625&$-10.62$\%\\
27 \hline
28 \end{tabular}\\
29 where the \% error is given by
30 \begin{equation}
31  \text{\% error} = \frac{\text{approx.} - \text{actual}}{\text{actual}}
32         = \frac{\alpha - \tan \alpha}{\tan \alpha}.
33 \end{equation}
34
35 So $31\dg$ is the largest whole-degree angle with $< 10$\% error.
36 \end{solution}
37